Inequality involving projectors on a Hilbert space isomorphic to $mathbb{C}^n$












1












$begingroup$


Suppose we have Hermitian operators $P_1$ and $P_2$ on Hilbert space $mathbb{H} cong mathbb{C}^n$, such that: $P_1^2=P_1$, $P_2^2=P_2$, ${P_1, P_2} = 0$. Let $x$ be a unit $L_2$-norm vector.



Show that $(x^TP_1x)^2 + (x^TP_2x)^2 leq 1$



I can only see how one can show that the same inequality holds for $RHS=2$. That can be easily seen because of the fact that a projection of a vector is always "shorter" than the vector. But I'm confused about the $RHS=1$ case.



I think I should somehow use the restriction on the anticommutator, but I can't figure that out. It is easy to prove that in this case $P_1+P_2$ is also a projector but I can't see how this helps here.










share|cite|improve this question











$endgroup$












  • $begingroup$
    What is the meaning of ${P_1,P_2}=0. $ ?
    $endgroup$
    – Fred
    Dec 17 '18 at 14:10










  • $begingroup$
    That's notation for an anticommutator, i.e. ${P_1, P_2}=P_1 P_2 + P_2 P_1$
    $endgroup$
    – Aleksandr Berezutskii
    Dec 17 '18 at 14:56


















1












$begingroup$


Suppose we have Hermitian operators $P_1$ and $P_2$ on Hilbert space $mathbb{H} cong mathbb{C}^n$, such that: $P_1^2=P_1$, $P_2^2=P_2$, ${P_1, P_2} = 0$. Let $x$ be a unit $L_2$-norm vector.



Show that $(x^TP_1x)^2 + (x^TP_2x)^2 leq 1$



I can only see how one can show that the same inequality holds for $RHS=2$. That can be easily seen because of the fact that a projection of a vector is always "shorter" than the vector. But I'm confused about the $RHS=1$ case.



I think I should somehow use the restriction on the anticommutator, but I can't figure that out. It is easy to prove that in this case $P_1+P_2$ is also a projector but I can't see how this helps here.










share|cite|improve this question











$endgroup$












  • $begingroup$
    What is the meaning of ${P_1,P_2}=0. $ ?
    $endgroup$
    – Fred
    Dec 17 '18 at 14:10










  • $begingroup$
    That's notation for an anticommutator, i.e. ${P_1, P_2}=P_1 P_2 + P_2 P_1$
    $endgroup$
    – Aleksandr Berezutskii
    Dec 17 '18 at 14:56
















1












1








1





$begingroup$


Suppose we have Hermitian operators $P_1$ and $P_2$ on Hilbert space $mathbb{H} cong mathbb{C}^n$, such that: $P_1^2=P_1$, $P_2^2=P_2$, ${P_1, P_2} = 0$. Let $x$ be a unit $L_2$-norm vector.



Show that $(x^TP_1x)^2 + (x^TP_2x)^2 leq 1$



I can only see how one can show that the same inequality holds for $RHS=2$. That can be easily seen because of the fact that a projection of a vector is always "shorter" than the vector. But I'm confused about the $RHS=1$ case.



I think I should somehow use the restriction on the anticommutator, but I can't figure that out. It is easy to prove that in this case $P_1+P_2$ is also a projector but I can't see how this helps here.










share|cite|improve this question











$endgroup$




Suppose we have Hermitian operators $P_1$ and $P_2$ on Hilbert space $mathbb{H} cong mathbb{C}^n$, such that: $P_1^2=P_1$, $P_2^2=P_2$, ${P_1, P_2} = 0$. Let $x$ be a unit $L_2$-norm vector.



Show that $(x^TP_1x)^2 + (x^TP_2x)^2 leq 1$



I can only see how one can show that the same inequality holds for $RHS=2$. That can be easily seen because of the fact that a projection of a vector is always "shorter" than the vector. But I'm confused about the $RHS=1$ case.



I think I should somehow use the restriction on the anticommutator, but I can't figure that out. It is easy to prove that in this case $P_1+P_2$ is also a projector but I can't see how this helps here.







linear-algebra projection projection-matrices






share|cite|improve this question















share|cite|improve this question













share|cite|improve this question




share|cite|improve this question








edited Dec 17 '18 at 14:18









mechanodroid

28.8k62548




28.8k62548










asked Dec 17 '18 at 14:00









Aleksandr BerezutskiiAleksandr Berezutskii

103




103












  • $begingroup$
    What is the meaning of ${P_1,P_2}=0. $ ?
    $endgroup$
    – Fred
    Dec 17 '18 at 14:10










  • $begingroup$
    That's notation for an anticommutator, i.e. ${P_1, P_2}=P_1 P_2 + P_2 P_1$
    $endgroup$
    – Aleksandr Berezutskii
    Dec 17 '18 at 14:56




















  • $begingroup$
    What is the meaning of ${P_1,P_2}=0. $ ?
    $endgroup$
    – Fred
    Dec 17 '18 at 14:10










  • $begingroup$
    That's notation for an anticommutator, i.e. ${P_1, P_2}=P_1 P_2 + P_2 P_1$
    $endgroup$
    – Aleksandr Berezutskii
    Dec 17 '18 at 14:56


















$begingroup$
What is the meaning of ${P_1,P_2}=0. $ ?
$endgroup$
– Fred
Dec 17 '18 at 14:10




$begingroup$
What is the meaning of ${P_1,P_2}=0. $ ?
$endgroup$
– Fred
Dec 17 '18 at 14:10












$begingroup$
That's notation for an anticommutator, i.e. ${P_1, P_2}=P_1 P_2 + P_2 P_1$
$endgroup$
– Aleksandr Berezutskii
Dec 17 '18 at 14:56






$begingroup$
That's notation for an anticommutator, i.e. ${P_1, P_2}=P_1 P_2 + P_2 P_1$
$endgroup$
– Aleksandr Berezutskii
Dec 17 '18 at 14:56












2 Answers
2






active

oldest

votes


















1












$begingroup$

In fact, the correct statement should be$$
x^TP_1x + x^TP_2x leq 1,quadcdots(*).
$$
(But your version is still true as we can see by squaring $(*)$.) This is called Bessel's inequality. To prove this, let
$$
x= P_1x + P_2 x + P_3x,$$

where $P_3 = I-P_1-P_2$. We can see that $P_3$ is also a projection and $P_iP_j = O$ for $ineq j$. Thus, we have
$$
1=x^Tx =sum_{i=1}^3 x^TP_ix +sum_{ineq j}x^TP_iP_jx= sum_{i=1}^3 x^TP_ix geq x^TP_1x + x^TP_2x,
$$
as desired.



$textbf{EDIT:}$ I added short proof that if $P,Q$ are orthogonal projections such that $PQ+QP=0$, then $PQ=QP=0$.



Assume to the contrary that $PQneq 0$. Then there is $v$ such that $v=Qv$ and $PQv=Pvneq 0$. By the assumption that $PQ=-QP$, we have
$$
Pv = -QPv.
$$
This implies that $Q$ has $-1$ as an eigenvalue. This leads to a contradiction.






share|cite|improve this answer











$endgroup$





















    0












    $begingroup$

    The condition $P_1P_2 + P_2P_1 = 0$ holds if and only if $P_1$ and $P_2$ project onto orthogonal subspaces. Also it is equivalent to $P_1 + P_2$ being an orthogonal projection.



    Notice that for an orthogonal projection $P$ and $x in Bbb{H}$ we have
    $$|Px|^2 = langle Px,Pxrangle = langle P^*Px,xrangle = langle P^2x,xrangle = langle Px,xrangle$$



    Hence for $x in Bbb{H}$, $|x| = 1$ we have



    begin{align}
    LHS &= langle P_1x,xrangle^2 + langle P_2x,xrangle^2 \
    &= |P_1x|^4 + |P_2x|^4 \
    &le |P_1x|^4 + 2|P_1x|^2|P_2x|^2 + |P_2x|^4 \
    &= (|P_1x|^2+|P_2x|^2)^2
    end{align}



    Since $P_1x perp P_2x$ we have



    $$LHS le (|P_1x|^2+|P_2x|^2)^2 = |P_1x + P_2x|^4 = |(P_1+P_2)x|^4 le |x|^4 = 1$$



    because $P_1 + P_2$ is an orthogonal projection and hence of norm $le 1$.






    share|cite|improve this answer









    $endgroup$













      Your Answer





      StackExchange.ifUsing("editor", function () {
      return StackExchange.using("mathjaxEditing", function () {
      StackExchange.MarkdownEditor.creationCallbacks.add(function (editor, postfix) {
      StackExchange.mathjaxEditing.prepareWmdForMathJax(editor, postfix, [["$", "$"], ["\\(","\\)"]]);
      });
      });
      }, "mathjax-editing");

      StackExchange.ready(function() {
      var channelOptions = {
      tags: "".split(" "),
      id: "69"
      };
      initTagRenderer("".split(" "), "".split(" "), channelOptions);

      StackExchange.using("externalEditor", function() {
      // Have to fire editor after snippets, if snippets enabled
      if (StackExchange.settings.snippets.snippetsEnabled) {
      StackExchange.using("snippets", function() {
      createEditor();
      });
      }
      else {
      createEditor();
      }
      });

      function createEditor() {
      StackExchange.prepareEditor({
      heartbeatType: 'answer',
      autoActivateHeartbeat: false,
      convertImagesToLinks: true,
      noModals: true,
      showLowRepImageUploadWarning: true,
      reputationToPostImages: 10,
      bindNavPrevention: true,
      postfix: "",
      imageUploader: {
      brandingHtml: "Powered by u003ca class="icon-imgur-white" href="https://imgur.com/"u003eu003c/au003e",
      contentPolicyHtml: "User contributions licensed under u003ca href="https://creativecommons.org/licenses/by-sa/3.0/"u003ecc by-sa 3.0 with attribution requiredu003c/au003e u003ca href="https://stackoverflow.com/legal/content-policy"u003e(content policy)u003c/au003e",
      allowUrls: true
      },
      noCode: true, onDemand: true,
      discardSelector: ".discard-answer"
      ,immediatelyShowMarkdownHelp:true
      });


      }
      });














      draft saved

      draft discarded


















      StackExchange.ready(
      function () {
      StackExchange.openid.initPostLogin('.new-post-login', 'https%3a%2f%2fmath.stackexchange.com%2fquestions%2f3043978%2finequality-involving-projectors-on-a-hilbert-space-isomorphic-to-mathbbcn%23new-answer', 'question_page');
      }
      );

      Post as a guest















      Required, but never shown

























      2 Answers
      2






      active

      oldest

      votes








      2 Answers
      2






      active

      oldest

      votes









      active

      oldest

      votes






      active

      oldest

      votes









      1












      $begingroup$

      In fact, the correct statement should be$$
      x^TP_1x + x^TP_2x leq 1,quadcdots(*).
      $$
      (But your version is still true as we can see by squaring $(*)$.) This is called Bessel's inequality. To prove this, let
      $$
      x= P_1x + P_2 x + P_3x,$$

      where $P_3 = I-P_1-P_2$. We can see that $P_3$ is also a projection and $P_iP_j = O$ for $ineq j$. Thus, we have
      $$
      1=x^Tx =sum_{i=1}^3 x^TP_ix +sum_{ineq j}x^TP_iP_jx= sum_{i=1}^3 x^TP_ix geq x^TP_1x + x^TP_2x,
      $$
      as desired.



      $textbf{EDIT:}$ I added short proof that if $P,Q$ are orthogonal projections such that $PQ+QP=0$, then $PQ=QP=0$.



      Assume to the contrary that $PQneq 0$. Then there is $v$ such that $v=Qv$ and $PQv=Pvneq 0$. By the assumption that $PQ=-QP$, we have
      $$
      Pv = -QPv.
      $$
      This implies that $Q$ has $-1$ as an eigenvalue. This leads to a contradiction.






      share|cite|improve this answer











      $endgroup$


















        1












        $begingroup$

        In fact, the correct statement should be$$
        x^TP_1x + x^TP_2x leq 1,quadcdots(*).
        $$
        (But your version is still true as we can see by squaring $(*)$.) This is called Bessel's inequality. To prove this, let
        $$
        x= P_1x + P_2 x + P_3x,$$

        where $P_3 = I-P_1-P_2$. We can see that $P_3$ is also a projection and $P_iP_j = O$ for $ineq j$. Thus, we have
        $$
        1=x^Tx =sum_{i=1}^3 x^TP_ix +sum_{ineq j}x^TP_iP_jx= sum_{i=1}^3 x^TP_ix geq x^TP_1x + x^TP_2x,
        $$
        as desired.



        $textbf{EDIT:}$ I added short proof that if $P,Q$ are orthogonal projections such that $PQ+QP=0$, then $PQ=QP=0$.



        Assume to the contrary that $PQneq 0$. Then there is $v$ such that $v=Qv$ and $PQv=Pvneq 0$. By the assumption that $PQ=-QP$, we have
        $$
        Pv = -QPv.
        $$
        This implies that $Q$ has $-1$ as an eigenvalue. This leads to a contradiction.






        share|cite|improve this answer











        $endgroup$
















          1












          1








          1





          $begingroup$

          In fact, the correct statement should be$$
          x^TP_1x + x^TP_2x leq 1,quadcdots(*).
          $$
          (But your version is still true as we can see by squaring $(*)$.) This is called Bessel's inequality. To prove this, let
          $$
          x= P_1x + P_2 x + P_3x,$$

          where $P_3 = I-P_1-P_2$. We can see that $P_3$ is also a projection and $P_iP_j = O$ for $ineq j$. Thus, we have
          $$
          1=x^Tx =sum_{i=1}^3 x^TP_ix +sum_{ineq j}x^TP_iP_jx= sum_{i=1}^3 x^TP_ix geq x^TP_1x + x^TP_2x,
          $$
          as desired.



          $textbf{EDIT:}$ I added short proof that if $P,Q$ are orthogonal projections such that $PQ+QP=0$, then $PQ=QP=0$.



          Assume to the contrary that $PQneq 0$. Then there is $v$ such that $v=Qv$ and $PQv=Pvneq 0$. By the assumption that $PQ=-QP$, we have
          $$
          Pv = -QPv.
          $$
          This implies that $Q$ has $-1$ as an eigenvalue. This leads to a contradiction.






          share|cite|improve this answer











          $endgroup$



          In fact, the correct statement should be$$
          x^TP_1x + x^TP_2x leq 1,quadcdots(*).
          $$
          (But your version is still true as we can see by squaring $(*)$.) This is called Bessel's inequality. To prove this, let
          $$
          x= P_1x + P_2 x + P_3x,$$

          where $P_3 = I-P_1-P_2$. We can see that $P_3$ is also a projection and $P_iP_j = O$ for $ineq j$. Thus, we have
          $$
          1=x^Tx =sum_{i=1}^3 x^TP_ix +sum_{ineq j}x^TP_iP_jx= sum_{i=1}^3 x^TP_ix geq x^TP_1x + x^TP_2x,
          $$
          as desired.



          $textbf{EDIT:}$ I added short proof that if $P,Q$ are orthogonal projections such that $PQ+QP=0$, then $PQ=QP=0$.



          Assume to the contrary that $PQneq 0$. Then there is $v$ such that $v=Qv$ and $PQv=Pvneq 0$. By the assumption that $PQ=-QP$, we have
          $$
          Pv = -QPv.
          $$
          This implies that $Q$ has $-1$ as an eigenvalue. This leads to a contradiction.







          share|cite|improve this answer














          share|cite|improve this answer



          share|cite|improve this answer








          edited Dec 17 '18 at 15:33

























          answered Dec 17 '18 at 14:19









          SongSong

          18.3k21549




          18.3k21549























              0












              $begingroup$

              The condition $P_1P_2 + P_2P_1 = 0$ holds if and only if $P_1$ and $P_2$ project onto orthogonal subspaces. Also it is equivalent to $P_1 + P_2$ being an orthogonal projection.



              Notice that for an orthogonal projection $P$ and $x in Bbb{H}$ we have
              $$|Px|^2 = langle Px,Pxrangle = langle P^*Px,xrangle = langle P^2x,xrangle = langle Px,xrangle$$



              Hence for $x in Bbb{H}$, $|x| = 1$ we have



              begin{align}
              LHS &= langle P_1x,xrangle^2 + langle P_2x,xrangle^2 \
              &= |P_1x|^4 + |P_2x|^4 \
              &le |P_1x|^4 + 2|P_1x|^2|P_2x|^2 + |P_2x|^4 \
              &= (|P_1x|^2+|P_2x|^2)^2
              end{align}



              Since $P_1x perp P_2x$ we have



              $$LHS le (|P_1x|^2+|P_2x|^2)^2 = |P_1x + P_2x|^4 = |(P_1+P_2)x|^4 le |x|^4 = 1$$



              because $P_1 + P_2$ is an orthogonal projection and hence of norm $le 1$.






              share|cite|improve this answer









              $endgroup$


















                0












                $begingroup$

                The condition $P_1P_2 + P_2P_1 = 0$ holds if and only if $P_1$ and $P_2$ project onto orthogonal subspaces. Also it is equivalent to $P_1 + P_2$ being an orthogonal projection.



                Notice that for an orthogonal projection $P$ and $x in Bbb{H}$ we have
                $$|Px|^2 = langle Px,Pxrangle = langle P^*Px,xrangle = langle P^2x,xrangle = langle Px,xrangle$$



                Hence for $x in Bbb{H}$, $|x| = 1$ we have



                begin{align}
                LHS &= langle P_1x,xrangle^2 + langle P_2x,xrangle^2 \
                &= |P_1x|^4 + |P_2x|^4 \
                &le |P_1x|^4 + 2|P_1x|^2|P_2x|^2 + |P_2x|^4 \
                &= (|P_1x|^2+|P_2x|^2)^2
                end{align}



                Since $P_1x perp P_2x$ we have



                $$LHS le (|P_1x|^2+|P_2x|^2)^2 = |P_1x + P_2x|^4 = |(P_1+P_2)x|^4 le |x|^4 = 1$$



                because $P_1 + P_2$ is an orthogonal projection and hence of norm $le 1$.






                share|cite|improve this answer









                $endgroup$
















                  0












                  0








                  0





                  $begingroup$

                  The condition $P_1P_2 + P_2P_1 = 0$ holds if and only if $P_1$ and $P_2$ project onto orthogonal subspaces. Also it is equivalent to $P_1 + P_2$ being an orthogonal projection.



                  Notice that for an orthogonal projection $P$ and $x in Bbb{H}$ we have
                  $$|Px|^2 = langle Px,Pxrangle = langle P^*Px,xrangle = langle P^2x,xrangle = langle Px,xrangle$$



                  Hence for $x in Bbb{H}$, $|x| = 1$ we have



                  begin{align}
                  LHS &= langle P_1x,xrangle^2 + langle P_2x,xrangle^2 \
                  &= |P_1x|^4 + |P_2x|^4 \
                  &le |P_1x|^4 + 2|P_1x|^2|P_2x|^2 + |P_2x|^4 \
                  &= (|P_1x|^2+|P_2x|^2)^2
                  end{align}



                  Since $P_1x perp P_2x$ we have



                  $$LHS le (|P_1x|^2+|P_2x|^2)^2 = |P_1x + P_2x|^4 = |(P_1+P_2)x|^4 le |x|^4 = 1$$



                  because $P_1 + P_2$ is an orthogonal projection and hence of norm $le 1$.






                  share|cite|improve this answer









                  $endgroup$



                  The condition $P_1P_2 + P_2P_1 = 0$ holds if and only if $P_1$ and $P_2$ project onto orthogonal subspaces. Also it is equivalent to $P_1 + P_2$ being an orthogonal projection.



                  Notice that for an orthogonal projection $P$ and $x in Bbb{H}$ we have
                  $$|Px|^2 = langle Px,Pxrangle = langle P^*Px,xrangle = langle P^2x,xrangle = langle Px,xrangle$$



                  Hence for $x in Bbb{H}$, $|x| = 1$ we have



                  begin{align}
                  LHS &= langle P_1x,xrangle^2 + langle P_2x,xrangle^2 \
                  &= |P_1x|^4 + |P_2x|^4 \
                  &le |P_1x|^4 + 2|P_1x|^2|P_2x|^2 + |P_2x|^4 \
                  &= (|P_1x|^2+|P_2x|^2)^2
                  end{align}



                  Since $P_1x perp P_2x$ we have



                  $$LHS le (|P_1x|^2+|P_2x|^2)^2 = |P_1x + P_2x|^4 = |(P_1+P_2)x|^4 le |x|^4 = 1$$



                  because $P_1 + P_2$ is an orthogonal projection and hence of norm $le 1$.







                  share|cite|improve this answer












                  share|cite|improve this answer



                  share|cite|improve this answer










                  answered Dec 17 '18 at 14:17









                  mechanodroidmechanodroid

                  28.8k62548




                  28.8k62548






























                      draft saved

                      draft discarded




















































                      Thanks for contributing an answer to Mathematics Stack Exchange!


                      • Please be sure to answer the question. Provide details and share your research!

                      But avoid



                      • Asking for help, clarification, or responding to other answers.

                      • Making statements based on opinion; back them up with references or personal experience.


                      Use MathJax to format equations. MathJax reference.


                      To learn more, see our tips on writing great answers.




                      draft saved


                      draft discarded














                      StackExchange.ready(
                      function () {
                      StackExchange.openid.initPostLogin('.new-post-login', 'https%3a%2f%2fmath.stackexchange.com%2fquestions%2f3043978%2finequality-involving-projectors-on-a-hilbert-space-isomorphic-to-mathbbcn%23new-answer', 'question_page');
                      }
                      );

                      Post as a guest















                      Required, but never shown





















































                      Required, but never shown














                      Required, but never shown












                      Required, but never shown







                      Required, but never shown

































                      Required, but never shown














                      Required, but never shown












                      Required, but never shown







                      Required, but never shown







                      Popular posts from this blog

                      Plaza Victoria

                      In PowerPoint, is there a keyboard shortcut for bulleted / numbered list?

                      How to put 3 figures in Latex with 2 figures side by side and 1 below these side by side images but in...